Show that $(a+b+c)^3 = a^3 + b^3 + c^3+ (a+b+c)(ab+ac+bc)$












15














As stated in the title, I'm supposed to show that $(a+b+c)^3 = a^3 + b^3 + c^3 + (a+b+c)(ab+ac+bc)$.



My reasoning:
$$(a + b + c)^3 = [(a + b) + c]^3 = (a + b)^3 + 3(a + b)^2c + 3(a + b)c^2 + c^3$$



$$(a + b + c)^3 = (a^3 + 3a^2b + 3ab^2 + b^3) + 3(a^2 + 2ab + b^2)c + 3(a + b)c^2+ c^3$$



$$(a + b + c)^3 = a^3 + b^3 + c^3 + 3a^2b + 3a^2c + 3ab^2 + 3b^2c + 3ac^2 + 3bc^2 + 6abc$$



$$(a + b + c)^3 = (a^3 + b^3 + c^3) + (3a^2b + 3a^2c + 3abc) + (3ab^2 + 3b^2c + 3abc) + (3ac^2 + 3bc^2 + 3abc) - 3abc$$



$$(a + b + c)^3 = (a^3 + b^3 + c^3) + 3a(ab + ac + bc) + 3b(ab + bc + ac) + 3c(ac + bc + ab) - 3abc$$



$$(a + b + c)^3 = (a^3 + b^3 + c^3) + 3(a + b + c)(ab + ac + bc) - 3abc$$



$$(a + b + c)^3 = (a^3 + b^3 + c^3) + 3[(a + b + c)(ab + ac + bc) - abc]$$
It doesn't look like I made careless mistakes, so I'm wondering if the statement asked is correct at all.










share|cite|improve this question




















  • 23




    The statement is obviously wrong: look at it when $a=b=c=1$.
    – KCd
    Jan 28 '13 at 14:43










  • You've probably not mentioned some condition?
    – hjpotter92
    Jan 28 '13 at 14:45










  • I transcribed the exercise. Found a typo or another at other parts of the book, so that's just another one of them.
    – Sawyier
    Jan 28 '13 at 14:48










  • If it helps the final statement in your derivation looks correct.
    – kaine
    Jan 28 '13 at 15:12






  • 2




    Incorrect questions teach you to figure out if a statement is true before wasting time trying to prove it.
    – DanielV
    Jul 20 '14 at 2:58
















15














As stated in the title, I'm supposed to show that $(a+b+c)^3 = a^3 + b^3 + c^3 + (a+b+c)(ab+ac+bc)$.



My reasoning:
$$(a + b + c)^3 = [(a + b) + c]^3 = (a + b)^3 + 3(a + b)^2c + 3(a + b)c^2 + c^3$$



$$(a + b + c)^3 = (a^3 + 3a^2b + 3ab^2 + b^3) + 3(a^2 + 2ab + b^2)c + 3(a + b)c^2+ c^3$$



$$(a + b + c)^3 = a^3 + b^3 + c^3 + 3a^2b + 3a^2c + 3ab^2 + 3b^2c + 3ac^2 + 3bc^2 + 6abc$$



$$(a + b + c)^3 = (a^3 + b^3 + c^3) + (3a^2b + 3a^2c + 3abc) + (3ab^2 + 3b^2c + 3abc) + (3ac^2 + 3bc^2 + 3abc) - 3abc$$



$$(a + b + c)^3 = (a^3 + b^3 + c^3) + 3a(ab + ac + bc) + 3b(ab + bc + ac) + 3c(ac + bc + ab) - 3abc$$



$$(a + b + c)^3 = (a^3 + b^3 + c^3) + 3(a + b + c)(ab + ac + bc) - 3abc$$



$$(a + b + c)^3 = (a^3 + b^3 + c^3) + 3[(a + b + c)(ab + ac + bc) - abc]$$
It doesn't look like I made careless mistakes, so I'm wondering if the statement asked is correct at all.










share|cite|improve this question




















  • 23




    The statement is obviously wrong: look at it when $a=b=c=1$.
    – KCd
    Jan 28 '13 at 14:43










  • You've probably not mentioned some condition?
    – hjpotter92
    Jan 28 '13 at 14:45










  • I transcribed the exercise. Found a typo or another at other parts of the book, so that's just another one of them.
    – Sawyier
    Jan 28 '13 at 14:48










  • If it helps the final statement in your derivation looks correct.
    – kaine
    Jan 28 '13 at 15:12






  • 2




    Incorrect questions teach you to figure out if a statement is true before wasting time trying to prove it.
    – DanielV
    Jul 20 '14 at 2:58














15












15








15


7





As stated in the title, I'm supposed to show that $(a+b+c)^3 = a^3 + b^3 + c^3 + (a+b+c)(ab+ac+bc)$.



My reasoning:
$$(a + b + c)^3 = [(a + b) + c]^3 = (a + b)^3 + 3(a + b)^2c + 3(a + b)c^2 + c^3$$



$$(a + b + c)^3 = (a^3 + 3a^2b + 3ab^2 + b^3) + 3(a^2 + 2ab + b^2)c + 3(a + b)c^2+ c^3$$



$$(a + b + c)^3 = a^3 + b^3 + c^3 + 3a^2b + 3a^2c + 3ab^2 + 3b^2c + 3ac^2 + 3bc^2 + 6abc$$



$$(a + b + c)^3 = (a^3 + b^3 + c^3) + (3a^2b + 3a^2c + 3abc) + (3ab^2 + 3b^2c + 3abc) + (3ac^2 + 3bc^2 + 3abc) - 3abc$$



$$(a + b + c)^3 = (a^3 + b^3 + c^3) + 3a(ab + ac + bc) + 3b(ab + bc + ac) + 3c(ac + bc + ab) - 3abc$$



$$(a + b + c)^3 = (a^3 + b^3 + c^3) + 3(a + b + c)(ab + ac + bc) - 3abc$$



$$(a + b + c)^3 = (a^3 + b^3 + c^3) + 3[(a + b + c)(ab + ac + bc) - abc]$$
It doesn't look like I made careless mistakes, so I'm wondering if the statement asked is correct at all.










share|cite|improve this question















As stated in the title, I'm supposed to show that $(a+b+c)^3 = a^3 + b^3 + c^3 + (a+b+c)(ab+ac+bc)$.



My reasoning:
$$(a + b + c)^3 = [(a + b) + c]^3 = (a + b)^3 + 3(a + b)^2c + 3(a + b)c^2 + c^3$$



$$(a + b + c)^3 = (a^3 + 3a^2b + 3ab^2 + b^3) + 3(a^2 + 2ab + b^2)c + 3(a + b)c^2+ c^3$$



$$(a + b + c)^3 = a^3 + b^3 + c^3 + 3a^2b + 3a^2c + 3ab^2 + 3b^2c + 3ac^2 + 3bc^2 + 6abc$$



$$(a + b + c)^3 = (a^3 + b^3 + c^3) + (3a^2b + 3a^2c + 3abc) + (3ab^2 + 3b^2c + 3abc) + (3ac^2 + 3bc^2 + 3abc) - 3abc$$



$$(a + b + c)^3 = (a^3 + b^3 + c^3) + 3a(ab + ac + bc) + 3b(ab + bc + ac) + 3c(ac + bc + ab) - 3abc$$



$$(a + b + c)^3 = (a^3 + b^3 + c^3) + 3(a + b + c)(ab + ac + bc) - 3abc$$



$$(a + b + c)^3 = (a^3 + b^3 + c^3) + 3[(a + b + c)(ab + ac + bc) - abc]$$
It doesn't look like I made careless mistakes, so I'm wondering if the statement asked is correct at all.







algebra-precalculus






share|cite|improve this question















share|cite|improve this question













share|cite|improve this question




share|cite|improve this question








edited Jul 20 '14 at 1:57









Cameron Williams

22.3k43679




22.3k43679










asked Jan 28 '13 at 14:37









SawyierSawyier

199118




199118








  • 23




    The statement is obviously wrong: look at it when $a=b=c=1$.
    – KCd
    Jan 28 '13 at 14:43










  • You've probably not mentioned some condition?
    – hjpotter92
    Jan 28 '13 at 14:45










  • I transcribed the exercise. Found a typo or another at other parts of the book, so that's just another one of them.
    – Sawyier
    Jan 28 '13 at 14:48










  • If it helps the final statement in your derivation looks correct.
    – kaine
    Jan 28 '13 at 15:12






  • 2




    Incorrect questions teach you to figure out if a statement is true before wasting time trying to prove it.
    – DanielV
    Jul 20 '14 at 2:58














  • 23




    The statement is obviously wrong: look at it when $a=b=c=1$.
    – KCd
    Jan 28 '13 at 14:43










  • You've probably not mentioned some condition?
    – hjpotter92
    Jan 28 '13 at 14:45










  • I transcribed the exercise. Found a typo or another at other parts of the book, so that's just another one of them.
    – Sawyier
    Jan 28 '13 at 14:48










  • If it helps the final statement in your derivation looks correct.
    – kaine
    Jan 28 '13 at 15:12






  • 2




    Incorrect questions teach you to figure out if a statement is true before wasting time trying to prove it.
    – DanielV
    Jul 20 '14 at 2:58








23




23




The statement is obviously wrong: look at it when $a=b=c=1$.
– KCd
Jan 28 '13 at 14:43




The statement is obviously wrong: look at it when $a=b=c=1$.
– KCd
Jan 28 '13 at 14:43












You've probably not mentioned some condition?
– hjpotter92
Jan 28 '13 at 14:45




You've probably not mentioned some condition?
– hjpotter92
Jan 28 '13 at 14:45












I transcribed the exercise. Found a typo or another at other parts of the book, so that's just another one of them.
– Sawyier
Jan 28 '13 at 14:48




I transcribed the exercise. Found a typo or another at other parts of the book, so that's just another one of them.
– Sawyier
Jan 28 '13 at 14:48












If it helps the final statement in your derivation looks correct.
– kaine
Jan 28 '13 at 15:12




If it helps the final statement in your derivation looks correct.
– kaine
Jan 28 '13 at 15:12




2




2




Incorrect questions teach you to figure out if a statement is true before wasting time trying to prove it.
– DanielV
Jul 20 '14 at 2:58




Incorrect questions teach you to figure out if a statement is true before wasting time trying to prove it.
– DanielV
Jul 20 '14 at 2:58










3 Answers
3






active

oldest

votes


















14














In general, $$a^n+b^n+c^n = sum_{i+2j+3k=n} frac{n}{i+j+k}binom {i+j+k}{i,j,k} s_1^i(-s_2)^js_3^k$$



where $s_1=a+b+c$, $s_2=ab+ac+bc$ and $s_3=abc$ are the elementary symmetric polynomials.



In the case that $n=3$, the triples possible are $(i,j,k)=(3,0,0),(1,1,0),$ and $(0,0,1)$ yielding the formula:



$$a^3+b^3+c^2 = s_1^3 - 3s_2s_1 + 3s_3$$



which is the result you got.



In general, any symmetric homogeneous polynomial $p(a,b,c)$ of degree $n$ can be written in the form:



$$p(a,b,c)=sum_{i+2j+3k=n} a_{i,j,k} s_1^i s_2^j s_3^k$$



for some constants $a_{i,j,k}$.



I've often thought Fermat's Last Theorem was most interesting when stated as a question about these polynomials. One statement of Fermat can be written as:




If $p$ is an odd prime, then $a^p+b^p+c^p=0$ if and only if $a+b+c=0$ and $abc=0$.







share|cite|improve this answer



















  • 1




    Regarding the last statement, shouldn't the three $n$s in the exponents be $p$s?
    – wythagoras
    Jan 30 '16 at 18:58



















1














$(a+b+c)^3=a^3+b^3+c^3+3(a+b)(b+c)(a+c)$ with $3(a+b)(b+c)(c+a) = 3a^2b + 3a^2c + 3ab^2 + 3b^2c + 3ac^2 + 3bc^2 + 6abc$



I guess the right factorization






share|cite|improve this answer































    1














    $(a+b+c)^3 = a^3 + b^3 + c^3 + 3(a+b+c)(ab+ac+bc) - 3abc$ is the right factorisation






    share|cite|improve this answer























    • You have a missing factor of $3$ on the right.
      – Start wearing purple
      Sep 9 '13 at 23:11










    protected by user99914 Oct 17 '15 at 4:20



    Thank you for your interest in this question.
    Because it has attracted low-quality or spam answers that had to be removed, posting an answer now requires 10 reputation on this site (the association bonus does not count).



    Would you like to answer one of these unanswered questions instead?














    3 Answers
    3






    active

    oldest

    votes








    3 Answers
    3






    active

    oldest

    votes









    active

    oldest

    votes






    active

    oldest

    votes









    14














    In general, $$a^n+b^n+c^n = sum_{i+2j+3k=n} frac{n}{i+j+k}binom {i+j+k}{i,j,k} s_1^i(-s_2)^js_3^k$$



    where $s_1=a+b+c$, $s_2=ab+ac+bc$ and $s_3=abc$ are the elementary symmetric polynomials.



    In the case that $n=3$, the triples possible are $(i,j,k)=(3,0,0),(1,1,0),$ and $(0,0,1)$ yielding the formula:



    $$a^3+b^3+c^2 = s_1^3 - 3s_2s_1 + 3s_3$$



    which is the result you got.



    In general, any symmetric homogeneous polynomial $p(a,b,c)$ of degree $n$ can be written in the form:



    $$p(a,b,c)=sum_{i+2j+3k=n} a_{i,j,k} s_1^i s_2^j s_3^k$$



    for some constants $a_{i,j,k}$.



    I've often thought Fermat's Last Theorem was most interesting when stated as a question about these polynomials. One statement of Fermat can be written as:




    If $p$ is an odd prime, then $a^p+b^p+c^p=0$ if and only if $a+b+c=0$ and $abc=0$.







    share|cite|improve this answer



















    • 1




      Regarding the last statement, shouldn't the three $n$s in the exponents be $p$s?
      – wythagoras
      Jan 30 '16 at 18:58
















    14














    In general, $$a^n+b^n+c^n = sum_{i+2j+3k=n} frac{n}{i+j+k}binom {i+j+k}{i,j,k} s_1^i(-s_2)^js_3^k$$



    where $s_1=a+b+c$, $s_2=ab+ac+bc$ and $s_3=abc$ are the elementary symmetric polynomials.



    In the case that $n=3$, the triples possible are $(i,j,k)=(3,0,0),(1,1,0),$ and $(0,0,1)$ yielding the formula:



    $$a^3+b^3+c^2 = s_1^3 - 3s_2s_1 + 3s_3$$



    which is the result you got.



    In general, any symmetric homogeneous polynomial $p(a,b,c)$ of degree $n$ can be written in the form:



    $$p(a,b,c)=sum_{i+2j+3k=n} a_{i,j,k} s_1^i s_2^j s_3^k$$



    for some constants $a_{i,j,k}$.



    I've often thought Fermat's Last Theorem was most interesting when stated as a question about these polynomials. One statement of Fermat can be written as:




    If $p$ is an odd prime, then $a^p+b^p+c^p=0$ if and only if $a+b+c=0$ and $abc=0$.







    share|cite|improve this answer



















    • 1




      Regarding the last statement, shouldn't the three $n$s in the exponents be $p$s?
      – wythagoras
      Jan 30 '16 at 18:58














    14












    14








    14






    In general, $$a^n+b^n+c^n = sum_{i+2j+3k=n} frac{n}{i+j+k}binom {i+j+k}{i,j,k} s_1^i(-s_2)^js_3^k$$



    where $s_1=a+b+c$, $s_2=ab+ac+bc$ and $s_3=abc$ are the elementary symmetric polynomials.



    In the case that $n=3$, the triples possible are $(i,j,k)=(3,0,0),(1,1,0),$ and $(0,0,1)$ yielding the formula:



    $$a^3+b^3+c^2 = s_1^3 - 3s_2s_1 + 3s_3$$



    which is the result you got.



    In general, any symmetric homogeneous polynomial $p(a,b,c)$ of degree $n$ can be written in the form:



    $$p(a,b,c)=sum_{i+2j+3k=n} a_{i,j,k} s_1^i s_2^j s_3^k$$



    for some constants $a_{i,j,k}$.



    I've often thought Fermat's Last Theorem was most interesting when stated as a question about these polynomials. One statement of Fermat can be written as:




    If $p$ is an odd prime, then $a^p+b^p+c^p=0$ if and only if $a+b+c=0$ and $abc=0$.







    share|cite|improve this answer














    In general, $$a^n+b^n+c^n = sum_{i+2j+3k=n} frac{n}{i+j+k}binom {i+j+k}{i,j,k} s_1^i(-s_2)^js_3^k$$



    where $s_1=a+b+c$, $s_2=ab+ac+bc$ and $s_3=abc$ are the elementary symmetric polynomials.



    In the case that $n=3$, the triples possible are $(i,j,k)=(3,0,0),(1,1,0),$ and $(0,0,1)$ yielding the formula:



    $$a^3+b^3+c^2 = s_1^3 - 3s_2s_1 + 3s_3$$



    which is the result you got.



    In general, any symmetric homogeneous polynomial $p(a,b,c)$ of degree $n$ can be written in the form:



    $$p(a,b,c)=sum_{i+2j+3k=n} a_{i,j,k} s_1^i s_2^j s_3^k$$



    for some constants $a_{i,j,k}$.



    I've often thought Fermat's Last Theorem was most interesting when stated as a question about these polynomials. One statement of Fermat can be written as:




    If $p$ is an odd prime, then $a^p+b^p+c^p=0$ if and only if $a+b+c=0$ and $abc=0$.








    share|cite|improve this answer














    share|cite|improve this answer



    share|cite|improve this answer








    edited May 11 '16 at 3:45









    Community

    1




    1










    answered Jan 28 '13 at 15:30









    Thomas AndrewsThomas Andrews

    130k11146297




    130k11146297








    • 1




      Regarding the last statement, shouldn't the three $n$s in the exponents be $p$s?
      – wythagoras
      Jan 30 '16 at 18:58














    • 1




      Regarding the last statement, shouldn't the three $n$s in the exponents be $p$s?
      – wythagoras
      Jan 30 '16 at 18:58








    1




    1




    Regarding the last statement, shouldn't the three $n$s in the exponents be $p$s?
    – wythagoras
    Jan 30 '16 at 18:58




    Regarding the last statement, shouldn't the three $n$s in the exponents be $p$s?
    – wythagoras
    Jan 30 '16 at 18:58











    1














    $(a+b+c)^3=a^3+b^3+c^3+3(a+b)(b+c)(a+c)$ with $3(a+b)(b+c)(c+a) = 3a^2b + 3a^2c + 3ab^2 + 3b^2c + 3ac^2 + 3bc^2 + 6abc$



    I guess the right factorization






    share|cite|improve this answer




























      1














      $(a+b+c)^3=a^3+b^3+c^3+3(a+b)(b+c)(a+c)$ with $3(a+b)(b+c)(c+a) = 3a^2b + 3a^2c + 3ab^2 + 3b^2c + 3ac^2 + 3bc^2 + 6abc$



      I guess the right factorization






      share|cite|improve this answer


























        1












        1








        1






        $(a+b+c)^3=a^3+b^3+c^3+3(a+b)(b+c)(a+c)$ with $3(a+b)(b+c)(c+a) = 3a^2b + 3a^2c + 3ab^2 + 3b^2c + 3ac^2 + 3bc^2 + 6abc$



        I guess the right factorization






        share|cite|improve this answer














        $(a+b+c)^3=a^3+b^3+c^3+3(a+b)(b+c)(a+c)$ with $3(a+b)(b+c)(c+a) = 3a^2b + 3a^2c + 3ab^2 + 3b^2c + 3ac^2 + 3bc^2 + 6abc$



        I guess the right factorization







        share|cite|improve this answer














        share|cite|improve this answer



        share|cite|improve this answer








        edited Jan 26 '15 at 17:36









        Debanjan Basu

        1869




        1869










        answered Jul 20 '14 at 1:51









        Quang VũQuang Vũ

        111




        111























            1














            $(a+b+c)^3 = a^3 + b^3 + c^3 + 3(a+b+c)(ab+ac+bc) - 3abc$ is the right factorisation






            share|cite|improve this answer























            • You have a missing factor of $3$ on the right.
              – Start wearing purple
              Sep 9 '13 at 23:11
















            1














            $(a+b+c)^3 = a^3 + b^3 + c^3 + 3(a+b+c)(ab+ac+bc) - 3abc$ is the right factorisation






            share|cite|improve this answer























            • You have a missing factor of $3$ on the right.
              – Start wearing purple
              Sep 9 '13 at 23:11














            1












            1








            1






            $(a+b+c)^3 = a^3 + b^3 + c^3 + 3(a+b+c)(ab+ac+bc) - 3abc$ is the right factorisation






            share|cite|improve this answer














            $(a+b+c)^3 = a^3 + b^3 + c^3 + 3(a+b+c)(ab+ac+bc) - 3abc$ is the right factorisation







            share|cite|improve this answer














            share|cite|improve this answer



            share|cite|improve this answer








            edited 2 days ago









            Confuse

            509513




            509513










            answered Sep 9 '13 at 22:44









            Carlos PeterCarlos Peter

            111




            111












            • You have a missing factor of $3$ on the right.
              – Start wearing purple
              Sep 9 '13 at 23:11


















            • You have a missing factor of $3$ on the right.
              – Start wearing purple
              Sep 9 '13 at 23:11
















            You have a missing factor of $3$ on the right.
            – Start wearing purple
            Sep 9 '13 at 23:11




            You have a missing factor of $3$ on the right.
            – Start wearing purple
            Sep 9 '13 at 23:11





            protected by user99914 Oct 17 '15 at 4:20



            Thank you for your interest in this question.
            Because it has attracted low-quality or spam answers that had to be removed, posting an answer now requires 10 reputation on this site (the association bonus does not count).



            Would you like to answer one of these unanswered questions instead?



            Popular posts from this blog

            Mario Kart Wii

            What does “Dominus providebit” mean?

            Antonio Litta Visconti Arese